\documentclass[12pt]{amsart} \usepackage{amssymb} \usepackage{amsmath} \usepackage{amsthm} \usepackage[all]{xy} \voffset=-1in \hoffset=-1in \textheight=10in \textwidth=6in \begin{document} \author{} \title{Math 772\\Spring 2011\\Homework 2} \date{} \maketitle \thispagestyle{empty} \pagestyle{empty} \centerline{\it Due Monday October 2, 2011.} \centerline{} \noindent 1. Find all primes $p$ such that: \begin{itemize} \item[a)] (5 pt) $-7$ is a square $\text{mod}(p)$. \item[b)] (5 pt) $\frac{2}{5}$ is a square $\text{mod}(p)$. \end{itemize} \centerline{} \noindent 2. (5 pt) Show that if $p$ is an odd prime then the units of $\mathbb{Z}/p^n\mathbb{Z}$ form a cyclic group. What happens in the case that $p=2$ (what is the group structure of $U(\mathbb{Z}/2^n\mathbb{Z})$)? \centerline{} \noindent 3. (5 pt) Consider the family of quadratic rings of integers $R:=\mathbb{Z}[\omega]$ where $\omega$ is given by \[ \omega= \begin{cases} \sqrt{d},\text{ if }d\equiv 2,3\text{ mod}(4);\\ \frac{1+\sqrt{d}}{2},\text{ if }d\equiv 1\text{ mod}(4). \end{cases} \] Show that if $R$ is an imaginary quadratic ring of integers ($d<0$), then $U(R)=\pm 1$ unless $d=-1$ or $d=-3$. What happens in these last two cases? By way of contrast, show that $U(\mathbb{Z}[\sqrt{2}])$ is infinite. \centerline{} \noindent 4. (5 pt) Let $R$ be an integral domain with quotient field $K$. We define the {\it integral closure} of $R$ to be \[ \overline{R}=\{\alpha\in K\vert p(\alpha)=0\text{ for some monic }p(x)\in R[x]\}. \] \noindent We say that $R$ is integrally closed if $R=\overline{R}$ (that is, $R$ already contains all of its integral elements from $K$). Prove that any UFD is integrally closed. \centerline{} \noindent 5. (5 pt) Let $R$ be an integral domain with quotient field $K$ and $p\in R$ a nonzero prime element. Show that $p$ is also a prime element of $R[x]$. \centerline{} \noindent 6. (5 pt) Let $F$ be a field extension of degree $n$ over $\mathbb{Q}$. Suppose that $\omega\in F$ is a root of a monic polynomial in $\mathbb{Z}[x]$. Show that $\omega$ is a root of a monic polynomial in $\mathbb{Z}[x]$ of degree no more than $n$ and, in particular, show that the minimal polynomial of $\omega$ (over $\mathbb{Q}$) may be taken to be monic and in $\mathbb{Z}[x]$. \centerline{} \noindent 7. (5 pt) Let $d$ be a square-free integer. Show that the ring of integers of the quadratic field $\mathbb{Q}(\sqrt{d})$ is given by \[ R=\begin{cases} \mathbb{Z}[\sqrt{d}] & \text{if $d\equiv 2,3\ \text{mod}(4)$,}\\ \mathbb{Z}[\frac{1+\sqrt{d}}{2}] & \text{if $d\equiv 1\ \text{mod}(4)$.} \end{cases} \] \end{document}